Você está na página 1de 6

AC TORT 2/2017 1

Instituto de Fı́sica
1o Q
UFRJ
2o Q
a
1 Avaliação a Distância de Fı́sica 3A - AD1
3o Q
Segundo Semestre de 2017 Bimestre 1
4o Q
Pólo : Data:
Nota
Nome :
Assinatura :

P ROBLEMA 1 Considere um elétron de massa


m e carga −e que descreve uma órbita circular
em torno de uma distribuição filiforme de carga
(isto é: em forma de fio) com uma densidade
linear uniforme λ > 0. O plano da órbita é per-
pendicular à distribuição e divide a mesma em
partes iguais (veja a figura). Calcule a magni- v
tude da velocidade orbital do elétron nos casos

2ℓ
seguintes:

(a) (10 pts) A distribuição se estende de −∞


até +∞;
λ
(b) (20 pts) A distribuição é finita e de com-
primento igual a 2 ℓ. Nesse caso, suponha
que o plano que contém a órbita do elétron
divide a distribuição em duas partes iguais.
Tome o limite ℓ → ∞ e compare o seu re-
sultado com o resultado anterior.

S OLUÇ ÃO 1:

(a) No limite em que o fio efetivamente é infinito podemos usar a lei de Gauss para calcular o campo:

1
E2πρ 2ℓ = λ 2ℓ;
ε0
logo,

λ
E= .
2πε0ρ
ou, vetorialmente
AC TORT 2/2017 2

λ
E= ρ̂.
2πε0ρ
onde ρ é a distância perpendicular ao fio. Segue que

eλ v2
− = −m ;
2πε0 ρ ρ
logo,
r

v= .
2πmε0
(b) Se o fio for finito, a lei de Gauss continua válida, mas não é uma boa ferramenta para calcular o
campo, elétrico pois agora E não pode ser colocado em evidência na integral que nos dá o fluxo.
Calculamos o campo em um ponto sobre o plano que corta o fio em duas partes iguais com a lei de
Coulomb e o prinı́cipio da superposição. Este cálculo você encontra em qualquer bom livro–texto
de Fı́sica III. O resultado é:

1 λℓ
E= p .
2πε0 ρ ρ2 + ℓ2
Segue que

1 e λℓ v2
p =m ;
2πε0 ρ ρ2 + ℓ2 ρ

1 e λℓ
v2 = p .
2πmε0 ρ2 + ℓ2
Observe que no limite ℓ → ∞ recuperamos o resultado do item anterior.

P ROBLEMA 2 Considere uma distribuição esférica de carga total Q0 descrita pela densidade volumar
de carga 
 0 x<a
ρ (r) = κ r2 a < r < b
0 r>b

onde r é a distância radial ao centro geométrico da distribuição.

(a) (10 pts) Determine o valor da constante κ.

(b) (20 pts) Determine o campo elétrico criado por esta distribuição em todo o espaço.
AC TORT 2/2017 3

S OLUÇ ÃO 2: Em todos os itens, dada a simetria esférica, usaremos:

Q(r)
E 4πr 2 = ;
ε0
onde
Z r
Q(r) = ρ(r ′ ) 4πr ′ 2 dr ′ .
a

(a)
b
4πκ 5
Z
r ′ 4 dr ′ = b − a5

Q0 = 4πκ
a 5
Deste resultado obtemos facilmente o valor de κ:

5Q0
κ= .
4π (b5 − a5 )
(b) Determine o campo elétrico criado por esta distribuição em todo o espaço.

Q(r)
E(r) = = 0; 0 ≤ r ≤ a.
4πε0 r 2
r
Q(r) 1
Z
E(r) = = κr ′ 2 4πr ′ 2 dr ′ ; a ≤ r ≤ b.
4πε0 r 2 4πε0 r 2 a
Efetuando a integral obtemos:

Q0 r 5 − a5
E(r) = ; a ≤ r ≤ b.
4πε0 r 2 b5 − a5
Por último,

b
Q(r) 1
Z
E(r) = = κr ′ 2 4πr ′ 2 dr ′ ; b ≤ r < ∞.
4πε0 r 2 4πεb r 2 a
O resultado agora é

Q0
E(r) = , b ≤ r < ∞,
4πε0 r 2
como esperado.
AC TORT 2/2017 4

P ROBLEMA 3 Quando o meio que circunda um condutor é o ar, o valor máximo do campo elétrico
muito próximo à superfı́cie deste é 3 × 106 V/m, se esse valor for ultrapassado o campo elétrico associado
com o condutor ionizará o ar ao redor. Considere inicialmente um condutor esférico de raio a = 10 cm.
O condutor está conectado a uma fonte de alta tensão de valor V .

(a) (10 pts) Determine o valor crı́tico Vcrı́tico da tensão V para que o condutor mantenha a sua carga e não
descarregue no ar.

(b) (30 pts) O condutor esférico é mantido sob a tensão crı́tica, mas é envolto por uma casca esférica
condutora aterrada de raio igual ao dobro do raio do condutor. Calcule o valor do campo elétrico
próximo à superfı́cie do condutor interno.

a a

b b

V b
b

b
V = Vcrı́tico b
b

b b

(a) (b)

Figura 1: No esquema (a), a esfera condutora é mantida a um potencial fixo V em relação à terra (o ponto
negro); no esquema (b), uma casca condutora aterrada envolve a esfera e este é mantida no potencial
V = Vcrı́tico .

S OLUÇ ÃO 3:

(a)
Vc = Ec a = 3 × 106 V/m × 10 × 10−2 m = 3 × 105 V;
Q
onde Ec = ,
4πǫ0 a2
(b) Há dois modos de resolver este item do problema:

Solução I Para um ponto a uma distância radial r do centro geométrico do sistema tal que a ≤ r ≤ b:

r
Q′ dr ′
Z
V (r) − V (a) = − ;
4πǫ0 a r′ 2
AC TORT 2/2017 5

onde Q ′ é a carga na esfera interior. Observe que Q ′ não é necessariamente igual à carga obtida no
item anterior! Efetuando a integral obtemos:
 
Q′ 1 1
V (r) − V (a) = − ; a ≤ r ≤ b.
4πǫ0 r a
Para determinar Q ′ fazemos r = b e usamos as condições (de contorno) V (b) = 0 e V (a) = Vc :
 
Q′ 1 1
0 − Vc = − ;
4πǫ0 b a
segue que:

Q′ ab
= Vc .
4πǫ0 b−a
Se b ≫ a, vemos que

Q′ Q
≈ aVc = ,
4πǫ0 4πǫ0
que é o resultado do item anterior. Finalmente, reunindo os resultados podemos escrever para o
potencial entre as esferas a expressão:

ab Vc b
V (r) = Vc + − Vc .
(b − a) r b−a
O campo elétrico no domı́nio a < r < b é dado por:

dV (r) ab Vc
E(r) = − = , a < r < b.
dr (b − a) r 2
Fazendo r → a:

b/a
E(r) → E(a) = Vc .
b−a
Como a = 10 cm e b = 20 cm, b = 2a, logo

Vc
E(a) = 2 = DUAS VEZES O RESULTADO DO ITEM ( A ) !
a

Solução II Outro modo de resolver o problema é resolver a equação de Laplace para o potencial em
coordenadas esféricas:
 
2 1 ∂ 2 ∂V
∇ V = 0, r = 0.
r 2 ∂r ∂r
Integrando a primeira vez:

∂V
r2 = C1 ;
∂r
AC TORT 2/2017 6

e integrando uma segunda vez:

C1
V (r) = − + C2 .
r
Para determinar as constantes de integração C1 e C2 fazemos uso das condições de contorno: V (a) =
Vc e V (b) = 0. Segue então que

ab Vc a
V (r) − Vc .
(b − a) r b−a
Para mostrar que as duas soluções são equivalentes basta somar e subtrair o termo Vc a/(b − a), no
resultado para o potencial obtido anteriormente:

ab Vc b a a
V (r) = Vc + − Vc + Vc − Vc .
(b − a) r b−a b−a b−a
Segue que

ab Vc a
V (r) − Vc .
(b − a) r b−a
Naturalmente, não se espera que o aluno siga este caminho. Oriente-o para que possa resolver o
problema pelo caminho I.

Você também pode gostar